lunes, 27 de junio de 2016

Problema del día

Sea $n \ge 3$ un entero positivo, y $t_1, t_2, ..., t_n$ números reales positivos tales que: 
\[n^2 + 1 > \left( t_1 + t_2 + \cdots + t_n \right) \left( \frac{1}{t_1} + \frac{1}{t_2} + \cdots + \frac{1}{t_n} \right).\]
Pruebe que $t_i, t_j, t_k$ son longitudes de un tríangulo para todo $i,j,k$ con $1 \leq i < j < k \leq n$

No hay comentarios.:

Publicar un comentario